¿La existencia de un monopolo magnético rompe la forma covariante de las ecuaciones de Maxwell para potenciales?

La ausencia de cargas magnéticas se refleja en una de las ecuaciones fundamentales de Maxwell:

(1) división B = 0.
Esta ecuación nos permite introducir el concepto de potencial vectorial:
B = putrefacción A .
Usando este concepto, es posible expresar las ecuaciones de Maxwell en una elegante forma simétrica:
(2) 2 A 1 C 2 2 A t 2 = j ϵ 0 C 2 ,
(3) 2 ϕ 1 C 2 2 ϕ t 2 = ρ ϵ 0 .

Notando, ese vector A y escalar ϕ potenciales, así como la densidad de corriente eléctrica j y densidad de carga ρ , forman un 4-vector en el espacio-tiempo de Minkovsky. Por lo tanto, las ecuaciones de Maxwell se pueden expresar en forma covariante, usando d'Alembertian:

(4) m m A v = j v ϵ 0 .

Si existen monopolos magnéticos, la ecuación de Maxwell ( 1 ) se verá como:

división B = m 0 C ρ metro a gramo norte mi t .

Como la divergencia de B no es igual a cero, es imposible introducir el concepto de vector potencial. Por lo tanto, la ecuación en forma de ( 4 ) no será posible expresar.

Si B 0 , pero × B = 0 , podría asignar un potencial escalar. Supongo..
Pero × B 0 debido a la ecuación de Maxwell.
No necesariamente. i = 0 Hace el truco. Básicamente, si hay bucles, se aplica el potencial vectorial pero no escalar. Y lo contrario si fuentes/sumideros de carga magnética. Pero si ambos están juntos, entonces tenemos un problema. Lo que hace que todo sea interesante..
Pregunta relacionada hecha por OP en TP.SE: physics.stackexchange.com/q/27755/2451
@Sergio Puede que no. Vea mi pregunta aquí: physics.stackexchange.com/questions/395167/…

Respuestas (2)

Otra opción, además de modificar el potencial A m = ( A i , ϕ ) de alguna manera, es introducir otro 4-potencial C m = ( C i , ψ ) . Entonces el campo eléctrico y magnético están dados por

mi = × C A t ϕ
B = × A C t ψ

Puede encontrar más información sobre este enfoque de 2 potenciales aquí: http://arxiv.org/abs/math-ph/0203043

En mi opinión, la introducción de otros 4 potenciales es la forma más natural de resolver este problema en contraste con el enfoque de fibra de Dirac. Es interesante preguntar, ¿cómo cambiará como resultado el procedimiento de cuantificación del campo electromagnético?
Me pregunto si esto es análogo a los potenciales de Hansen para un espacio-tiempo que es estacionario pero no estático.

Sí, introducir un monopolo magnético en las ecuaciones de Maxwell significa que ya no es posible la existencia de un potencial vectorial que se define en todas partes y en todas partes es continuo. En particular, esto puede ser molesto porque la representación del vector potencial es crucial para la predicción del valor cuantificado de una carga magnética (tanto en la forma en que se desarrolló históricamente como en la forma en que generalmente se presenta).

Aunque no se ha afirmado la existencia del monopolo magnético, se han realizado muchas investigaciones sobre cómo solucionar este problema. La mayoría de las formulaciones en realidad se adhieren a alguna forma de vector potencial, ya que es el marco existente y es muy conveniente. Esto implica que dicho vector potencial compatible con monopolos se convierte en una bestia menos sencilla. No estoy seguro de cuál se considera la mejor autoridad en esto, pero Wikipedia dice sobre el tema (y coincide con mi propio entendimiento):

En la teoría matemática de los monopolos magnéticos, se permite que A sea indefinido o de valores múltiples en algunos lugares.

El tema fue abordado por primera vez por Dirac, y su posición se resume aquí :

El razonamiento de Dirac muestra que es consistente en la mecánica cuántica describir un monopolo magnético con el potencial vectorial Ecuación 3, aunque tenga una singularidad de "cuerda".

Sin embargo, el enfoque de Dirac está un poco desactualizado. Hoy en día, uno trataría el tema utilizando la teoría del haz de fibras.
Por cierto, un artículo relevante pero algo antiguo al respecto: TT Wu en CN Yang "Dirac Monopole without Strings Classical Lagrangian Theory", Physical Review D Vol 14-2 p437-445, 1976.